Difference between revisions of "The Limit Theorems for Functions"

From Department of Mathematics at UTSA
Jump to navigation Jump to search
Line 15: Line 15:
 
<li>Similarly, since <math>\lim_{x \to c} f(x) = M</math> then for <math>\epsilon_2 = \frac{\epsilon}{2}</math> <math>\exists \delta_2 > 0</math> such that if <math>x \in A</math> and <math>0 < \mid x - c \mid < \delta_2</math> then <math>\mid f(x) - M \mid < \epsilon_2 = \frac{\epsilon}{2}</math>. Now let <math>\delta = \mathrm{min} \{ \delta_1, \delta_2 \}</math> and so we have that:</li>
 
<li>Similarly, since <math>\lim_{x \to c} f(x) = M</math> then for <math>\epsilon_2 = \frac{\epsilon}{2}</math> <math>\exists \delta_2 > 0</math> such that if <math>x \in A</math> and <math>0 < \mid x - c \mid < \delta_2</math> then <math>\mid f(x) - M \mid < \epsilon_2 = \frac{\epsilon}{2}</math>. Now let <math>\delta = \mathrm{min} \{ \delta_1, \delta_2 \}</math> and so we have that:</li>
 
</ul>
 
</ul>
<math>\begin{align} \quad \quad \mid L - M \mid = \mid L - f(x) + f(x) - M \mid \mid L - f(x) \mid + \mid f(x) - M \mid < \epsilon_1 + \epsilon_2 = \frac{\epsilon}{2} + \frac{\epsilon}{2} = \epsilon \end{align}</math>
+
<math>\begin{align} \quad \quad \mid L - M \mid = \mid L - f(x) + f(x) - M \mid \leq \mid L - f(x) \mid + \mid f(x) - M \mid < \epsilon_1 + \epsilon_2 = \frac{\epsilon}{2} + \frac{\epsilon}{2} = \epsilon \end{align}</math>
 
<ul>
 
<ul>
 
<li>But <math>\epsilon > 0</math> is arbitrary, so this implies that <math>\mid L - M \mid = 0</math>, that is <math>L = M</math>, a contradiction. So our assumption that <math>L \neq M</math> was false, and so if <math>\lim_{x \to c} f(x) = L</math> then <math>L</math> is unique. <math>\blacksquare</math></li>
 
<li>But <math>\epsilon > 0</math> is arbitrary, so this implies that <math>\mid L - M \mid = 0</math>, that is <math>L = M</math>, a contradiction. So our assumption that <math>L \neq M</math> was false, and so if <math>\lim_{x \to c} f(x) = L</math> then <math>L</math> is unique. <math>\blacksquare</math></li>
 
</ul>
 
</ul>

Revision as of 15:12, 21 October 2021

The Uniqueness of Limits of a Function Theorem

Recall from <a href="/the-limit-of-a-function">The Limit of a Function</a> page that for a function where is a cluster point of , then if such that if and then . We have not yet established that the limit is unique, so is it possible that and where ? The following theorem will show that this cannot happen.

Theorem (Uniqueness of Limits): Let be a function and let be a cluster point of . Then if are both limits of at , that is and , then .
  • Proof: Let be a function and let be a cluster point of . Also let and . Suppose that . We will show that this leads to a contradiction. Let be given.
  • Since , then for such that if and then .
  • Similarly, since then for such that if and then . Now let and so we have that:

  • But is arbitrary, so this implies that , that is , a contradiction. So our assumption that was false, and so if then is unique.